Đến nội dung

Hình ảnh

Topic BẤT ĐẲNG THỨC ôn thi vào lớp 10 THPT 2017 - 2018

bất đẳng thức am-gm cauchy bunyakovski minskovski schwarz holder thcs

  • Please log in to reply
Chủ đề này có 299 trả lời

#101
Hoang Dinh Nhat

Hoang Dinh Nhat

    Sĩ quan

  • Thành viên
  • 402 Bài viết

Bài 40: (sưu tầm)

Cho các số thực dương a,b,c. Chứng minh rằng:

$(1+\frac{a}{b})(1+\frac{b}{c})(1+\frac{c}{a})\geq 2(1+\frac{a+b+c}{\sqrt[3]{abc}})$


Chấp nhận giới hạn của bản thân, nhưng đừng bao giờ bỏ cuộc

 

 

 

 


#102
AnhTran2911

AnhTran2911

    Thượng sĩ

  • Thành viên
  • 230 Bài viết

Lời giải bài 40:

ÁP DỤNG AM-GM 3 số dương:

$LHS=\sum({\frac{a}{b}+\frac{a}{c}+\frac{a}{a}}) -1\geq\sum\frac{3a}{\sqrt[3]{abc}}-1$

       $=\frac{3(a+b+c)}{\sqrt[3]{abc}}-1\geq 3-1+\frac{2(a+b+c)}{\sqrt[3]{abc}} = RHS$


Bài viết đã được chỉnh sửa nội dung bởi AnhTran2911: 02-05-2017 - 13:12

        AQ02

                                 


#103
AnhTran2911

AnhTran2911

    Thượng sĩ

  • Thành viên
  • 230 Bài viết

BÀI TOÁN 41( Sưu tầm):  Cho $a,b,c\ge0$ thỏa mãn $abc=1$

CMR: $\frac{1}{\sqrt{2a^2+6a+1}}+\frac{1}{\sqrt{2b^2+6b+1}}+\frac{1}{\sqrt{2c^2+6c+1}}\geq1$


        AQ02

                                 


#104
Drago

Drago

    Sĩ quan

  • Thành viên
  • 462 Bài viết

Bài 42: (Sưu tầm): Cho $\Delta ABC$ có chu vi bằng 1. Chứng minh rằng:

$ \frac{13}{27}\leq a^2+b^2+c^2+4abc< \frac{1}{2}$


$\mathbb{VTL}$


#105
AnhTran2911

AnhTran2911

    Thượng sĩ

  • Thành viên
  • 230 Bài viết

Dễ có $a,b,c\leq\frac{1}{2}$ . Do vậy $(1-2a)(1-2b)(1-2c)\geq0$ ( do k hông xảy ra TH cả 3 số đều bằng $\frac{1}{2}$)

$\Rightarrow 1-8abc-2(a+b+c) +4(ab+bc+ca)\ge0 = 1-8abc-2(a+b+c)^2+4(ab+bc+ca)\ge0$

:ukliam2: CM vế phải: sử dụng BĐT AM-GM cho 3 số dương ta có:

$(1-2a)(1-2b)(1-2c)\leq\frac{1}{27}$ ( Do $a+b+c=1$)

$\Rightarrow-8abc+4(ab+bc+ca)-2(a+b+c)\leq\frac{-26}{27}$

$\Rightarrow4abc+a+b+c-2(ab+bc+ca)\geq\frac{13}{27}$

$\Rightarrow4abc+(a+b+c)^2-2(ab+bc+ca)\geq\frac{13}{27}$

$\Rightarrow4abc+a^2+b^2+c^2\geq\frac{13}{27}$. ( ĐPCM).


Bài viết đã được chỉnh sửa nội dung bởi AnhTran2911: 03-05-2017 - 12:34

        AQ02

                                 


#106
TrBaoChis

TrBaoChis

    Hạ sĩ

  • Banned
  • 81 Bài viết
$BÀI 43$:(Sưu tầm)
Cho a,b,c > 0 thỏa $a^2$+$b^2$+$c^2$ $=$ 1
$CMR$ :
$\sum$ $\frac{a}{a^3+bc}$ $\geq$ 3  

Bài viết đã được chỉnh sửa nội dung bởi TrBaoChis: 03-05-2017 - 12:25


#107
AnhTran2911

AnhTran2911

    Thượng sĩ

  • Thành viên
  • 230 Bài viết

 

$BÀI 43$:(Sưu tầm)
Cho x,y,z > 0 thỏa $x^2$+$y^2$+$z^2$ $=$ 1
$CMR$ :
$\sum$ $\frac{a}{a^3+bc}$ $\geq$ 3  

 

CHO x,y đi CM a,b kìa

Mà hình như a,b,c không âm chứ


Bài viết đã được chỉnh sửa nội dung bởi AnhTran2911: 03-05-2017 - 12:31

        AQ02

                                 


#108
haccau

haccau

    Binh nhất

  • Thành viên
  • 47 Bài viết

BÀI 44: (ASM-CHUYÊN TOÁN - 2015)

Cho ba số thực dương a,b,c thỏa mãn (a+b)(b+c)(c+a)=1. Chứng minh:

                           $ab+bc+ca\leq \frac{3}{4}$


Bài viết đã được chỉnh sửa nội dung bởi haccau: 03-05-2017 - 17:31

:lol:  :lol:  :lol: Don't let your dreams just be dreams!!! :lol:  :lol:  :lol: 


#109
AnhTran2911

AnhTran2911

    Thượng sĩ

  • Thành viên
  • 230 Bài viết

BÀI 44: (ASM-CHUYÊN TOÁN - 2015)

Cho ba số thực dương a,b,c thỏa mãn (a+b)(b+c)(c+a)=1. Chứng minh:

$ab+bc+ca\leq \frac{3}{4}$

Áp dụng BĐT $(a+b)(b+c)(c+a)\geq\frac{8}{9}(a+b+c)(ab+bc+ca)\geq\frac{8}{9}\sqrt{3(ab+bc+ca)}(ab+bc+ca)$

Từ đây có đpcm.

PS: Có ai làm đc bài số 41 của em chưa ạ?


Bài viết đã được chỉnh sửa nội dung bởi AnhTran2911: 03-05-2017 - 17:42

        AQ02

                                 


#110
Nguyenhuyen_AG

Nguyenhuyen_AG

    Trung úy

  • Thành viên nổi bật 2016
  • 945 Bài viết

BÀI TOÁN 41( Sưu tầm):  Cho $a,b,c\ge0$ thỏa mãn $abc=1$

CMR: $\frac{1}{\sqrt{2a^2+6a+1}}+\frac{1}{\sqrt{2b^2+6b+1}}+\frac{1}{\sqrt{2c^2+6c+1}}\geq1$

 

Chỉ cần chứng minh

\[\frac{1}{\sqrt{2a^2+6a+1}} \geqslant \frac{1}{a^{10/9} + a^{5/9} + 1}.\]


Nguyen Van Huyen
Ho Chi Minh City University Of Transport

#111
tuaneee111

tuaneee111

    Trung sĩ

  • Thành viên
  • 174 Bài viết

tại sao anh tìm đc đánh giá trên vậy?


$$\boxed{\boxed{I\heartsuit MATHEMATICAL}}$$

Blog của tôi

:luoi: Sức hấp dẫn của toán học mãnh liệt đến nỗi tôi bắt đầu sao nhãng các môn học khác - Sofia Vasilyevna Kovalevskaya :lol:


#112
AnhTran2911

AnhTran2911

    Thượng sĩ

  • Thành viên
  • 230 Bài viết

Chỉ cần chứng minh

\[\frac{1}{\sqrt{2a^2+6a+1}} \geqslant \frac{1}{a^{10/9} + a^{5/9} + 1}.\]

Có cách giải nào ngoài phương pháp Đ giá đại diện sau khi dùng đạo hàm như anh k ạ?

Cách này em biết rồi nhưng ở đây ta cần một lời giải phù hợp THCS hơn ạ. 

Ps: chỉ là góp ý nho nhỏ,


        AQ02

                                 


#113
Nguyenhuyen_AG

Nguyenhuyen_AG

    Trung úy

  • Thành viên nổi bật 2016
  • 945 Bài viết

tại sao anh tìm đc đánh giá trên vậy?

 

Anh dùng hệ số bất định kết hợp với đạo hàm.

 

Có cách giải nào ngoài phương pháp Đ giá đại diện sau khi dùng đạo hàm như anh k ạ?

Cách này em biết rồi nhưng ở đây ta cần một lời giải phù hợp THCS hơn ạ. 

Ps: chỉ là góp ý nho nhỏ,

 

Đổi biến $(a,b,c) \to \left(\frac{bc}{a^2},\frac{ca}{b^2},\frac{ab}{c^2}\right)$ và đánh giá bằng bất đẳng thức Holder

\[\left(\sum \frac{a^2}{\sqrt{a^4+6a^2bc+2b^2c^2}}\right)^2 \sum a^2(a^4+6a^2bc+2b^2c^2) \geqslant (a^2+b^2+c^2)^3.\]


Nguyen Van Huyen
Ho Chi Minh City University Of Transport

#114
Mr Cooper

Mr Cooper

    Sĩ quan

  • Thành viên
  • 496 Bài viết

$\boxed{\textbf{Bài Toán 45}}$ $\text{[IMO 2001]}$ Cho $a,b,c$ là các số thực dương. Chứng minh rằng:

\[\frac{a}{\sqrt{a^2+8bc}}+\frac{b}{\sqrt{b^2+8ca}}+\frac{c}{\sqrt{c^2+8ab}} \ge 1\]


Bài viết đã được chỉnh sửa nội dung bởi Mr Cooper: 04-05-2017 - 11:04


#115
AnhTran2911

AnhTran2911

    Thượng sĩ

  • Thành viên
  • 230 Bài viết

$\boxed{\textbf{Bài Toán 45}}$ $\text{[IMO 2001]}$ Cho $a,b,c$ là các số thực dương. Chứng minh rằng:

\[\frac{a}{\sqrt{a^2+8bc}}+\frac{b}{\sqrt{b^2+8ca}}+\frac{c}{\sqrt{c^2+8ab}} \ge 1\]

Theo CS thì $LHS=\sum\frac{a^2}{a\sqrt{a^2+8bc}}\geq\frac{(a+b+c)^2}{\sum{a}\sqrt{a^2+8bc}}\geq$

                           $\frac{(a+b+c)^2}{\sqrt{(a+b+c)(\sum{a^3}+24abc)}}\geq$ $\frac{(a+b+c)^2}{\sqrt{(a+b+c)(a+b+c)^3}}=RHS$


Bài viết đã được chỉnh sửa nội dung bởi AnhTran2911: 04-05-2017 - 11:20

        AQ02

                                 


#116
NHoang1608

NHoang1608

    Sĩ quan

  • Thành viên
  • 375 Bài viết

Bài toán 46: (Albania TST 2012)

   Tìm giá trị lớn nhất của biểu thức 

                                            $\frac{1}{x^{2}-4x+9}+\frac{1}{y^{2}-4y+9}+\frac{1}{z^{2}-4z+9}$

   Trong đó $x,y,z$ là các số thực không âm thỏa mãn $x+y+z=1$.


The greatest danger for most of us is not that our aim is too high and we miss it, but that it is too low and we reach it.

----- Michelangelo----


#117
tuaneee111

tuaneee111

    Trung sĩ

  • Thành viên
  • 174 Bài viết

$\boxed{\textbf{Bài Toán 45}}$ $\text{[IMO 2001]}$ Cho $a,b,c$ là các số thực dương. Chứng minh rằng:

\[\frac{a}{\sqrt{a^2+8bc}}+\frac{b}{\sqrt{b^2+8ca}}+\frac{c}{\sqrt{c^2+8ab}} \ge 1\]

Đặt VT=P . Áp dụng bất đẳng thức Holder ta có:

\[{P^2}\left( {\sum\limits_{cyc} {a\left( {{a^2} + 8bc} \right)} } \right) \ge {\left( {a + b + c} \right)^3}\]

Vậy cần chứng minh:  

\[\frac{{{{\left( {a + b + c} \right)}^3}}}{{\sum\limits_{cyc} {a\left( {{a^2} + 8bc} \right)} }} \ge 1 \Leftrightarrow ab\left( {a + b} \right) + bc\left( {b + c} \right) + ca\left( {c + a} \right) \ge 6abc\]

Bất đẳng thức cuối đúng theo AM-GM nên có điều phải chứng minh.

$$\boxed{\boxed{I\heartsuit MATHEMATICAL}}$$

Blog của tôi

:luoi: Sức hấp dẫn của toán học mãnh liệt đến nỗi tôi bắt đầu sao nhãng các môn học khác - Sofia Vasilyevna Kovalevskaya :lol:


#118
Chu Quang Huy

Chu Quang Huy

    Binh nhì

  • Thành viên mới
  • 12 Bài viết

Bài  46 (sưu tầm) Cho các số thực a,b,c >0 và abc=1.

      Chứng minh rằng :  (a2+b2+c2)≥ 9(a3+b3+c3)


Bài viết đã được chỉnh sửa nội dung bởi Chu Quang Huy: 05-05-2017 - 14:53


#119
Nguyenhuyen_AG

Nguyenhuyen_AG

    Trung úy

  • Thành viên nổi bật 2016
  • 945 Bài viết

Bài  46 (sưu tầm) Cho các số thực a,b,c >0 và abc=1.

      Chứng minh rằng :  (a2+b2+c2)≥ 9(a3+b3+c3)

 

Ta có

\[(a^2+b^2+c^2)^3 - 9(a^3+b^3+c^3) = \frac13\sum(ab+2bc+2ca+c^2)(a-b)^4+\frac12\sum\left[(a^2+b^2)^2+5c^4\right](a-b)^2.\]


Nguyen Van Huyen
Ho Chi Minh City University Of Transport

#120
Chu Quang Huy

Chu Quang Huy

    Binh nhì

  • Thành viên mới
  • 12 Bài viết

Ta có

\[(a^2+b^2+c^2)^3 - 9(a^3+b^3+c^3) = \frac13\sum(ab+2bc+2ca+c^2)(a-b)^4+\frac12\sum\left[(a^2+b^2)^2+5c^4\right](a-b)^2.\]

Aảo quá .Có ai còn cách nào tự nhiên hơn không?







Được gắn nhãn với một hoặc nhiều trong số những từ khóa sau: bất đẳng thức, am-gm, cauchy, bunyakovski, minskovski, schwarz, holder, thcs

1 người đang xem chủ đề

0 thành viên, 1 khách, 0 thành viên ẩn danh